0 Daumen
130 Aufrufe

Hi, kann mir jemand bei folgender Aufgabe helfen? 

Bei a) kann ich i) => iii) folgern, bzw. bei b) i)=> iii)

Kann mir jemand einen Hinweis geben was mir die Aussage iv) bringt?

Bin für jeden Hinweis dankbar.



Seien X, Y, Z Mengen und F : X → Y , G : Y → Z Abbildungen.

 a) Betrachten Sie die folgende Aussagen

 i) G ◦ F ist injektiv 

ii) F ist injektiv

 iii) G ist injektiv

 iv) G|F(x) : F(x) → Z ist injektiv. 

Geben Sie alle Implikationen zwischen Kombinationen der Aussagen i)-iv) an. Erklären Sie Ihre Antwort. 


b) Formulieren Sie dieselbe Aufgabe wie oben über Surjektivität der Verknüpfungen der Abbildungen und lösen Sie diese Aufgabe.

Avatar von

Ein anderes Problem?

Stell deine Frage

Willkommen bei der Mathelounge! Stell deine Frage einfach und kostenlos

x
Made by a lovely community